Projectile motion question totally stuck

In summary, the conversation discusses a problem involving a cannon firing projectiles up a hill at a given angle and how to calculate the maximum possible range. The suggested solution involves separating the initial velocity into horizontal and vertical components, using equations for constant speed or accelerated motion, and considering the angle of the hill's slope. The final solution involves solving a quadratic equation and using a derivative or recalling high school math to find the maximum range.
  • #1
emyt
217
0

Homework Statement



A cannon is arrange to fire projectiles, with initial speed V0, directly up the face of a hill of elevation angle alpha. At what angle from the horizontal should the cannon be aimed to obtain the maximum possible range R up the face of the hill?

Homework Equations



v = v0 + at?

x = x0 + v0t + at^2 ?


The Attempt at a Solution



I have no idea what to do, please help my self esteem is totally shot .. I can't believe that I actually have no clue
 
Physics news on Phys.org
  • #2
Just begin as you would any projectile problem.
Separate the initial velocity into horizontal and vertical parts. Then write two headings:
Horizontal and Vertical. Decide in each case whether you have constant speed or accelerated motion and write the appropriate formula(s).
You'll need some model for the hill - it is like a straight line on a y vs x graph so you know its equation.
 
  • #3
Delphi51 said:
Just begin as you would any projectile problem.
Separate the initial velocity into horizontal and vertical parts. Then write two headings:
Horizontal and Vertical. Decide in each case whether you have constant speed or accelerated motion and write the appropriate formula(s).
You'll need some model for the hill - it is like a straight line on a y vs x graph so you know its equation.

Hi, thanks for the reply - I tried doing something like..

Vx0 = V0costheta

Vy0 = V0sintheta

Rsinalpha = V0sintheta - 1/2gt^2

Rcosalpha = V0costheta
thanks a lot
 
Last edited:
  • #4
emyt said:
Hi, thanks for the reply - I tried doing something like..

Vx0 = V0costheta

Vy0 = V0sintheta

Rsinalpha = V0sintheta - 1/2gt^2

Rcosalpha = V0costheta

but this didn't get me anywhere.. are you telling me to treat the hill as if it were a flat surface? what do I do with the angle the hill makes then?

thanks a lot

Realign your coordinate system so that the positive x-axis is pointed up the slope. The angle of the slope comes into play when you calculate the accelerations in the x and y axes.
 
  • #5
RoyalCat said:
Realign your coordinate system so that the positive x-axis is pointed up the slope. The angle of the slope comes into play when you calculate the accelerations in the x and y axes.


can I just consider the hill as a straight line, calculate the angle from there and then just add on alpha after? I thought of this but I wasn't sure..

thanks
 
  • #6
Isn't the max for any equation 45 degrees
 
  • #7
45 degrees is the optimum angle to maximize the range on a flat surface. Throwing up a hill at angle alpha, the optimum angle will vary with alpha.

emyt, I was thinking that a hill of angle alpha will have a slope of tan(alpha) so its equation will be y = mx = tan(alpha)*x. [1]

For the horizontal part, constant speed, you'll have something like
x = v*cos(θ)*t [2]
For the vertical part, instead of "Rsinalpha = V0sintheta - 1/2gt^2 " I would just write y = v*sin(θ)*t - 1/2gt^2 in this notation. [3]
Sub [1] into [2] or [3] so you are looking at the situation when the projectile hits the hill. Study the remaining two equations and see if you can see what angle gives the maximum x or y that you are looking for.
 
  • #8
Delphi51 said:
45 degrees is the optimum angle to maximize the range on a flat surface. Throwing up a hill at angle alpha, the optimum angle will vary with alpha.

emyt, I was thinking that a hill of angle alpha will have a slope of tan(alpha) so its equation will be y = mx = tan(alpha)*x. [1]

For the horizontal part, constant speed, you'll have something like
x = v*cos(θ)*t [2]
For the vertical part, instead of "Rsinalpha = V0sintheta - 1/2gt^2 " I would just write y = v*sin(θ)*t - 1/2gt^2 in this notation. [3]
Sub [1] into [2] or [3] so you are looking at the situation when the projectile hits the hill. Study the remaining two equations and see if you can see what angle gives the maximum x or y that you are looking for.

hi, thanks for the reply:

I thought that I was looking for rcosalpha and rsinalpha as my final x and y.. since those would be the coordinates at the end of the range?

thanks
 
  • #9
Oh, I see it now! I was confused at the lack of x's and y's. You are saying the same thing I did and you have already got it down to two equations. Missing a couple of t's, though. It's x = v*t and
y = v*t + .5*a*t^2.

If you solve the easier one for t and sub in the other, you get a nice quadratic in r so you should be able to get its maximum either using a derivative or recalling a quadratic's max from high school math.
 
  • #10
well, to find a max range
cant we use R=u2sin2[tex]\theta[/tex]/g
 
  • #11
Delphi51 said:
Oh, I see it now! I was confused at the lack of x's and y's. You are saying the same thing I did and you have already got it down to two equations. Missing a couple of t's, though. It's x = v*t and
y = v*t + .5*a*t^2.

If you solve the easier one for t and sub in the other, you get a nice quadratic in r so you should be able to get its maximum either using a derivative or recalling a quadratic's max from high school math.


yeah I got it, thanks - I was repulsed by the hideous substituting... it wasn't so bad

thanks again
 
  • #12
That's a powerful little formula, look416!
But it is only for shooting on a horizontal surface and doesn't apply to this problem.

Congrats, emyt!
 
  • #13
hi, I tried putting -4.9(rcosalpha/V0x costheta)^2 + v0sintheta(rcosalpha/V0x costheta) - Rsinalpha = 0 into the quadratic formula and it got really messy :S should I be doing it like this? there are too many unknowns..

thank you
 
  • #14
oh i got it
thx for clearing my confusion
 
  • #15
I didn't get such a messy quadratic as you. Mine had no c term so I could common factor it. However, I ended up with a formula for tan θ that blows up when alpha equals zero, so I must have an error.

Perhaps if we work together on it we can find each other's errors.
You are supposed to take the lead. How about we use A for alpha and T for theta so it isn't quite so messy? Do you have
r*cos(A) = v*cos(T)*t and r*sin(A) = V*sin(T)*t - .5*g*t^2
as the starting point? What did you get next?
 
  • #16
Delphi51 said:
I didn't get such a messy quadratic as you. Mine had no c term so I could common factor it. However, I ended up with a formula for tan θ that blows up when alpha equals zero, so I must have an error.

Perhaps if we work together on it we can find each other's errors.
You are supposed to take the lead. How about we use A for alpha and T for theta so it isn't quite so messy? Do you have
r*cos(A) = v*cos(T)*t and r*sin(A) = V*sin(T)*t - .5*g*t^2
as the starting point? What did you get next?

hi, I had the same thing, when I solved for t I got rcos(A)/vcos(T)

then I plugged it into the sin equation and subtracted V*sin(T) from both sides to equate it to zero so I could find rcos(A)/vcos(T) and I thought that if I could assign a number to that, I could solve for theta.. but it didn't work

thanks for working with me
 
  • #17
Okay so putting t = rcos(A)/vcos(T) into r*sin(A) = V*sin(T)*t - .5*g*t^2
gives you
r*sin(A) = V*sin(T)*rcos(A)/vcos(T) - .5*g*r^2/v^2*cos^2(A)/cos^2(T)
Why now subtract V*sin(T) from both sides? Suggest we divide both sides by r and take the two terms that have no r to the left side.

This is fun - let's keep going!
 
  • #18
Delphi51 said:
Okay so putting t = rcos(A)/vcos(T) into r*sin(A) = V*sin(T)*t - .5*g*t^2
gives you
r*sin(A) = V*sin(T)*rcos(A)/vcos(T) - .5*g*r^2/v^2*cos^2(A)/cos^2(T)
Why now subtract V*sin(T) from both sides? Suggest we divide both sides by r and take the two terms that have no r to the left side.

This is fun - let's keep going!

I equated it to zero so I could try solving for rcos(A)/vcos(T), but I guess I could divide by r as well, I just thought it would work with that method from what you said before
thanks I'm on it
 
  • #19
Delphi51 said:
Okay so putting t = rcos(A)/vcos(T) into r*sin(A) = V*sin(T)*t - .5*g*t^2
gives you
r*sin(A) = V*sin(T)*rcos(A)/vcos(T) - .5*g*r^2/v^2*cos^2(A)/cos^2(T)
Why now subtract V*sin(T) from both sides? Suggest we divide both sides by r and take the two terms that have no r to the left side.

This is fun - let's keep going!

hi, what did you get? :S I'm doing so much expanding and whatnot it doesn't seem right
 
Last edited:
  • #20
sin(A) = V*sin(T)*cos(A)/vcos(T) - .5*g*r/v^2*cos^2(A)/cos^2(T)
after dividing by r.
I'll take the r term to the left and the sin(A) to the right:
.5*g*r/v^2*cos^2(A)/cos^2(T) = V*sin(T)*cos(A)/vcos(T) - sin(A)
How would you solve that for r?
 
  • #21
Delphi51 said:
sin(A) = V*sin(T)*cos(A)/vcos(T) - .5*g*r/v^2*cos^2(A)/cos^2(T)
after dividing by r.
I'll take the r term to the left and the sin(A) to the right:
.5*g*r/v^2*cos^2(A)/cos^2(T) = V*sin(T)*cos(A)/vcos(T) - sin(A)
How would you solve that for r?

oh right, thanks.. then you divide and multiply to isolate r..

r = (V*sin(T)*cos(A)/vcos(T) - sin(A)*[v^2*cos^2(A)/cos^2(T)])/.5*g

but V theta and alpha are all unknown?
 
  • #22
We must consider that alpha (A) is known - the specified hill angle.
Our job is to find the theta that makes r a maximum.
To avoid the mistake I made last time, we should check at this stage to see if it makes sense when alpha is zero.
 
  • #23
If you rotate your coordinate plane (I believe):
[tex]x = vocos(\theta - \alpha ) t - \frac{1}{2}gsin\alpha t^2[/tex]

[tex]y = vosin(\theta - \alpha) t -\frac{1}{2}gcos\alpha t^2[/tex]If you don't rotate your coordinate plane:
[tex] x = vocos\theta t [/tex]

[tex] y = vosin\theta t - \frac{1}{2}gt^2[/tex]

[tex] tan \alpha = \frac{y}{x}[/tex]

[tex] R = \sqrt{x^2+y^2}[/tex][tex]\theta = [/tex] Launch angle
[tex]\alpha = [/tex] Angle of incline
 
Last edited:
  • #24
Thanks, Jebus. Regret I don't understand your very first step, the
v*cos(theta)*t. It seems to me that after you rotate, your v would no longer be v but something like v*cos(alpha).

emyt, I'm not agreeing with your last step. I get
.5*g*r/v^2*cos^2(A)/cos^2(T) = V*sin(T)*cos(A)/vcos(T) - sin(A)
.5*g*r = V^2*sin(T)*cos^3(A)/cos^3(T) - v^2*sin(A)*cos^2(A)/cos^2(T)
and then
r = 2/g*V^2*sin(T)*cos^3(A)/cos^3(T) - 2/g*v^2*sin(A)*cos^2(A)/cos^2(T)
 
  • #25
I probably should've stated the variables. I'll edit that in :>
 
  • #26
Jebus_Chris said:
If you rotate your coordinate plane (I believe):
[tex]x = vocos(\theta - \alpha ) t - \frac{1}{2}gsin\alpha t^2[/tex]

[tex]y = vosin(\theta - \alpha) t -\frac{1}{2}gcos\alpha t^2[/tex]If you don't rotate your coordinate plane:
[tex] x = vocos\theta t [/tex]

[tex] y = vosin\theta t - \frac{1}{2}gt^2[/tex]

[tex] tan \alpha = \frac{y}{x}[/tex]

[tex] R = \sqrt{x^2+y^2}[/tex][tex]\theta = [/tex] Launch angle
[tex]\alpha = [/tex] Angle of incline

hi, if I don't rotate the coordinate plane, wouldn't x and y be Rcosalpha and Rsinalpha?
and I see how you get v0cos(theta - alpha) but how did you get gsinalphat^2 .. etc ?

thanks
 
Last edited:
  • #27
Delphi51 said:
Thanks, Jebus. Regret I don't understand your very first step, the
v*cos(theta)*t. It seems to me that after you rotate, your v would no longer be v but something like v*cos(alpha).

emyt, I'm not agreeing with your last step. I get
.5*g*r/v^2*cos^2(A)/cos^2(T) = V*sin(T)*cos(A)/vcos(T) - sin(A)
.5*g*r = V^2*sin(T)*cos^3(A)/cos^3(T) - v^2*sin(A)*cos^2(A)/cos^2(T)
and then
r = 2/g*V^2*sin(T)*cos^3(A)/cos^3(T) - 2/g*v^2*sin(A)*cos^2(A)/cos^2(T)

I think if you do the algebra it works out to be the same thing, I just copied and pasted thing around.. anyway, don';t you think that the answer should have a bit more of a concise method? there's point where I feel like the answer isn't going anywhere...
 
  • #28
emyt said:
hi, if I don't rotate the coordinate plane, wouldn't x and y be Rcosalpha and Rsinalpha?

thanks
Yep. That may require more manipulation of equations in the long run though.
 
  • #29
In the rotated coordinates, how do you get the time and x value when it hits the hill?
 
  • #30
Delphi51 said:
Thanks, Jebus. Regret I don't understand your very first step, the
v*cos(theta)*t. It seems to me that after you rotate, your v would no longer be v but something like v*cos(alpha).

emyt, I'm not agreeing with your last step. I get
.5*g*r/v^2*cos^2(A)/cos^2(T) = V*sin(T)*cos(A)/vcos(T) - sin(A)
.5*g*r = V^2*sin(T)*cos^3(A)/cos^3(T) - v^2*sin(A)*cos^2(A)/cos^2(T)
and then
r = 2/g*V^2*sin(T)*cos^3(A)/cos^3(T) - 2/g*v^2*sin(A)*cos^2(A)/cos^2(T)

were you planning to plug that in somewhere? that's humongous
 
  • #31
Found a mistake.
r = 2v^2/(g*cos^2(A)) times this trig expression:
cos(A)*sin(T)*cos(T) - sin(A)*cos^2(T)
Only this last expression varies with T, so only it matters.
To get the max for r, we differentiate this with respect to T and set it equal to zero. Almost there! I already checked when A=0 and got the 45 degrees we would expect!

Jebut, your rotated solution gets as messy as ours when you set y = 0 to be on the hill, solve for t and sub into the x equation.
 
  • #32
Delphi51 said:
Found a mistake.
r = 2v^2/(g*cos^2(A)) times this trig expression:
cos(A)*sin(T)*cos(T) - sin(A)*cos^2(T)
Only this last expression varies with T, so only it matters.
To get the max for r, we differentiate this with respect to T and set it equal to zero. Almost there! I already checked when A=0 and got the 45 degrees we would expect!

Jebut, your rotated solution gets as messy as ours when you set y = 0 to be on the hill, solve for t and sub into the x equation.

okay, thanks for checking that it works. let me check it out

so you got r to be 2v^2/(g*cos^2(A)) and which equation did you plug it into? either one of the x or y?

thanks a lot
 
  • #33
Well, if we forgo rotating, I think the best approach here would be to look at these two equations as intersecting curves/lines.

One is the regular quadratic for projectile motion, and the other would be the straight line [tex]y=\tan{\alpha}x[/tex]

Setting the two to be equal, and solving for the [tex]x[/tex] where they intersect will give us the [tex]x[/tex] coordinate of their intersection (And the trivial solution [tex]x=0[/tex])

Since that's a linear function, maximizing [tex]x_{impact}[/tex] would maximize the whole of the function. All we've got left to do from there is to differentiate with respect to [tex]\theta[/tex] and equate to zero.

Now it's just a question of getting over the algebra.
 
  • #34
RoyalCat said:
Well, if we forgo rotating, I think the best approach here would be to look at these two equations as intersecting curves/lines.

One is the regular quadratic for projectile motion, and the other would be the straight line [tex]y=\tan{\alpha}x[/tex]

Setting the two to be equal, and solving for the [tex]x[/tex] where they intersect will give us the [tex]x[/tex] coordinate of their intersection (And the trivial solution [tex]x=0[/tex])

Since that's a linear function, maximizing [tex]x_{impact}[/tex] would maximize the whole of the function. All we've got left to do from there is to differentiate with respect to [tex]\theta[/tex] and equate to zero.

Now it's just a question of getting over the algebra.

thanks, I was really thinking about the axis rotation.. I don't get why the answer isn't just pi/4 + alpha?

on a straight line, the maximum angle is pi/4 so if you turned the axis, it would be pi/4 + alpha?
 
  • #35
emyt said:
thanks, I was really thinking about the axis rotation.. I don't get why the answer isn't just pi/4 + alpha?

on a straight line, the maximum angle is pi/4 so if you turned the axis, it would be pi/4 + alpha?

anybody? I actually think this is a good idea, but where have I gone wrong?

thanks
 

Similar threads

Replies
2
Views
2K
Replies
3
Views
3K
  • Introductory Physics Homework Help
Replies
8
Views
2K
  • Introductory Physics Homework Help
Replies
18
Views
3K
  • Introductory Physics Homework Help
Replies
4
Views
1K
  • Introductory Physics Homework Help
Replies
7
Views
2K
  • Introductory Physics Homework Help
Replies
19
Views
2K
  • Introductory Physics Homework Help
Replies
3
Views
1K
  • Introductory Physics Homework Help
Replies
18
Views
2K
  • Introductory Physics Homework Help
Replies
19
Views
3K
Back
Top